QUESTION TEXT: One year ago a local government initiated an antismoking…
QUESTION TYPE: Strengthen
CONCLUSION: The anti-smoking advertising campaign did have an effect on the number of local people who smoke cigarettes.
REASONING: One year after the advertising campaign 3% less people smoke in the local area.
ANALYSIS: This argument makes a correlation-causation error. The campaign might have caused the decrease but there could have been another cause. For example, cigarette prices were raised by 20 cents. That may have deterred smokers from buying cigarettes.
We can strengthen this argument by eliminating alternative causes.
___________
- The conclusion is strictly about the number of people smoking cigarettes. Use of other tobacco products is irrelevant.
- The conclusion was about the number of people who smoke cigarettes. Any amount of cigarettes. The amount smoked is irrelevant.
- This may show that people are healthier. But it doesn’t help us establish that the campaign led to the decrease in smoking.
- CORRECT. This supports the argument by showing that consumers paid the same price for cigarettes even though a tax was placed on cigarettes. It eliminates an alternate explanation for the drop in smoking.
- This doesn’t let us conclude that high prices caused the decline in pricing. The smokers may have continued purchasing cigarettes even though the price was higher. This is consistent with the idea that it was the ad campaign that caused the decrease.
Recap: The question begins with “One year ago a local government initiated an antismoking”. It is a Strengthen question. Learn more about LSAT Strengthen questions in our guide to LSAT Logical Reasoning question types.
Nikhil says
Hey! I am having trouble understanding why A is incorrect. I do think D works but I can’t understand why A doesn’t.
Here, we are given a correlation between the ad, price increase and drop in smoking. From the correlation, we get a causation that the ad is what caused the drop in smoking.
The flaw here is that the author overlooks all other alternate causes of the drop in smoking. In a strengthen question, an AC that denies an alternate causal explanation wins. For example, an AC that says or implies that X, an alternate cause for the drop in smoking, did not actually happen or that it can’t be the cause will be the correct AC.
Coming to AC A which says that the residents did not increase use of other forms of tobacco. Here, X i.e. the alternate cause, is given as people’s increased use of other forms of tobacco. AC A denies this alternate cause.
In my head, the explanation that the 3% decrease in smoking happened because people switched over to other forms of tobacco seems like a valid alternate cause for the drop. It is such a small percentage and it is entirely reasonable that people switched how they wanted their tobacco kick. So, “3% people stopped smoking because they had switched over to other forms of tobacco instead” is a wonderful alternate causal explanation. Denying this alternate explnation increases teh liklihood of ad causing the drop being true.
Tutor Aaminah Qureshi says
I see your point, and you’re right that ruling out alternative explanations is a common way to strengthen an argument. However, the key issue here is proving that the ads caused the decline in smoking, not just that smoking declined.
Answer A states that residents didn’t switch to other tobacco products. While this might matter in another context (like overall health outcomes), it doesn’t directly strengthen the argument that the ads caused the decline in cigarette smoking.
Let’s consider if people DID switch to other forms of tobacco. The ads could still have caused the decline in cigarette smoking, as they were specifically antismoking ads. Even if Answer A said “Residents increased their use of other tobacco products …”, it could still support the idea that the ANTISMOKING ADS led to less cigarette use, just through a different mechanism (by making people switch over to other products).
This shows that whether or not people switched to other tobacco products is not what we’re looking for to prove that the ads caused the decline in cigarette smoking. The question asks for the option that MOST strengthens the argument, and D does this by showing that the decline wasn’t due to a price increase – something directly mentioned in the passage. Other tobacco products aren’t mentioned, and D is simply just a better answer than A.
Hope that clarifies things, and apologies for the late response!